Diễn Đàn MathScope

Diễn Đàn MathScope (http://forum.mathscope.org/index.php)
-   2012 (http://forum.mathscope.org/forumdisplay.php?f=175)
-   -   [IMO 2012] Bài 2 - Bất đẳng thức (http://forum.mathscope.org/showthread.php?t=33560)

novae 09-07-2012 09:49 PM

[IMO 2012] Bài 2 - Bất đẳng thức
 
Cho số nguyên $n \ge 3$ và các số thực dương $a_2,a_3,\ldots,a_n$ thỏa mãn $a_2 \cdots a_n= 1$. Chứng minh rằng
$$ (1+a_2)^2(1+a_3)^3 \cdots (1+a_n)^n > n^n $$

Harry Potter 11-07-2012 01:36 AM

Dùng cái này là xong mà $(a_k+1)=\left(a_k+\frac 1{k-1}+\cdots+\frac 1{k-1}\right)\geq k\sqrt[k]{\frac{a_k}{(k-1)^{k-1}}}$

Shuichi Akai 11-07-2012 01:56 AM

Mình làm giống như ý tưởng bạn ở trên :D
AM - GM cho k số:
$(1+a_k)^k=\left (\dfrac{1}{k-1}.(k-1)+a_k\right )^k\ge \dfrac{a_k}{(k-1)^{k-1}}.k^k $
từ đấy suy ra
$\prod (1+a_i)^i\geq a_2a_3...a_n\frac{2^2.3^3...n^n}{1^1.2^2...(n-1)^{n-1}}=n^n $
dấu bằng xảy ra $\Leftrightarrow a_i = \frac{1}{i-1} \Rightarrow a_2a_3...a_n < 1 $ (mâu thuẫn)
Vậy ta có đpcm :go:

AnhIsGod 11-07-2012 08:28 AM

Bài này chẳng phải AM-GM trực tiếp cho từng thừa số sau khi tách ra rồi xét dấu bằng thì là gì?

hamaianh0405 11-07-2012 09:27 AM

Vậy nó cho cái tích $a_{1}a_{2}...a_{n}=1 $ để làm gì vì dấu bằng mâu thuẫn

Toan95cqb 11-07-2012 10:34 AM

Trích:

Nguyên văn bởi hamaianh0405 (Post 161163)
Vậy nó cho cái tích $a_{1}a_{2}...a_{n}=1 $ để làm gì vì dấu bằng mâu thuẫn

Bài này không có dấu bằng. Bạn xem lại đề đi $>n^n $

hansongkyung 11-07-2012 08:31 PM

Một cách khác dựa trên ý tưởng trên
Vì $a_2a_3...a_n =1$ nên tồn tại các số $x_2,x_3,...,x_n$ sao cho:
$a_2=\frac{x_2}{x_3}, a_3=\frac{x_4}{x_3},...,a_n=\frac{x_n}{x_2}$
Sử dụng bđt AM-GM ta có
$(\frac{x_i+x_{i+1}}{x_{i+1}})^k \ge \frac{i^{i}}{(i-1)^{i-1}
}\frac{x_{i}}{x_{i+1}}$
Cho $i$ chạy từ $2 \to n$ ta có ĐPCM
:-h
Việt Nam cố lên

minhphuc.v 12-07-2012 09:55 PM

Dùng BĐT Holder ta có
$$(1+1)^1(1+a_2)^2(1+a_3)^3...(1+a_n)^n \ge (1+a_2.a_3...a_n)^{\frac{n(n+1)}{2}}$$
$$=2^{\frac{n(n+1)}{2}} = \left( 2^{\frac{(n+1)}{2}}\right)^n \ge ( 2\ln2.n )^n=n^n(2\ln2)^n > 2n^n, (n \ge 3)$$
Vì ta có hàm số $$f(x)=2^{\frac{x+1}{2}}-2\ln2.x \ge 0, \forall x \ge 3$$


Múi giờ GMT. Hiện tại là 11:38 AM.

Powered by: vBulletin Copyright ©2000-2024, Jelsoft Enterprises Ltd.

[page compression: 8.44 k/9.04 k (6.58%)]